Cutting down an Ideal

The name of the pictureThe name of the pictureThe name of the pictureClash Royale CLAN TAG#URR8PPP











up vote
2
down vote

favorite












Let $A subset B$ two commutative rings and $I subset A$ an Ideal. I shall write $I B$ for the Ideal in $B$ generated by $I$. Now, is it always true that $I = A cap I B$ ?



We used something like this in our lecture (however we did not prove it and used it only for Dedekind-Domains) and I was wondering if the statement holds for every commutative rings. The inclusion "$subset$" is trivial, but I don't know about "$supset$". Any help would be much appreciated.










share|cite|improve this question

























    up vote
    2
    down vote

    favorite












    Let $A subset B$ two commutative rings and $I subset A$ an Ideal. I shall write $I B$ for the Ideal in $B$ generated by $I$. Now, is it always true that $I = A cap I B$ ?



    We used something like this in our lecture (however we did not prove it and used it only for Dedekind-Domains) and I was wondering if the statement holds for every commutative rings. The inclusion "$subset$" is trivial, but I don't know about "$supset$". Any help would be much appreciated.










    share|cite|improve this question























      up vote
      2
      down vote

      favorite









      up vote
      2
      down vote

      favorite











      Let $A subset B$ two commutative rings and $I subset A$ an Ideal. I shall write $I B$ for the Ideal in $B$ generated by $I$. Now, is it always true that $I = A cap I B$ ?



      We used something like this in our lecture (however we did not prove it and used it only for Dedekind-Domains) and I was wondering if the statement holds for every commutative rings. The inclusion "$subset$" is trivial, but I don't know about "$supset$". Any help would be much appreciated.










      share|cite|improve this question













      Let $A subset B$ two commutative rings and $I subset A$ an Ideal. I shall write $I B$ for the Ideal in $B$ generated by $I$. Now, is it always true that $I = A cap I B$ ?



      We used something like this in our lecture (however we did not prove it and used it only for Dedekind-Domains) and I was wondering if the statement holds for every commutative rings. The inclusion "$subset$" is trivial, but I don't know about "$supset$". Any help would be much appreciated.







      abstract-algebra ring-theory commutative-algebra ideals






      share|cite|improve this question













      share|cite|improve this question











      share|cite|improve this question




      share|cite|improve this question










      asked 2 days ago









      user9620780

      775




      775




















          2 Answers
          2






          active

          oldest

          votes

















          up vote
          4
          down vote



          accepted










          $$A=Bbb ZsubsetBbb R=B;,;;I:=3Bbb Zimplies IB=IBbb R=Bbb R;,;;textso...$$






          share|cite|improve this answer




















          • Is there a general rule in what rings this statement holds? I know it works in Dedekind-Domains, and if $A = K[X]$ and $B=L[X]$ for two fields $K subset L$ it also works.
            – user9620780
            2 days ago


















          up vote
          2
          down vote













          DonAntonio gave a good counterexample, I just thought I'd try to make the problem of which rings have this property a bit easier by reducing it to a local question.



          The inclusion map $Ito Acap IB$ is surjective if and only if for all primes $newcommandppmathfrakpppinnewcommandSpecoperatornameSpecSpec A$ the localized map $I_ppto (Acap IB)_pp=A_ppcap (IB)_pp=A_ppcap I_pp B_pp$ is surjective. The first equality is proved here, and the second is true, since if $I$ is an ideal in a commutative ring, $A$, $S$ is a multiplicative set, and $M$ is an $A$ module, it's fairly clear that $S^-1IM subseteq (S^-1I)(S^-1M)$, and conversely if $sum_i fraca_is_ifracm_it_i in (S^-1I)(S^-1M)$, multiplying by $prod_i s_it_i$ gives an element of $IM$, so the original element of $(S^-1I)(S^-1M)$ must have been an element of $S^-1IM$.



          Hence to show that $I=Acap IB$ for all ideals $I$ in $A$ it is sufficient to prove the property locally. I.e., that for all points $pp$ of $Spec A$ and ideals $I_ppsubset A_pp$ we have $I_pp=A_ppcap I_pp B_pp$.



          In fact it suffices to just prove this for the maximal ideals. See page 7 here or any standard text on commutative algebra.



          It's not too hard to show that this property holds for the maximal ideals in a Dedekind domain, (with $B$ integral over $A$ ofc) since $A_pp$ is a DVR (when $pp$ is maximal). Thus we have that the ideals of $A_pp$ are $(pi^n)$ for $pi$ a uniformizer for $A_pp$, and if $pi^n-1in(pi^n)B_pp$, we would have $pi^n-1=pi^n b$ for some $bin B_pp$, or $1=pi b$, but $frac1pi$ cannot possibly be integral over $A_pp$, since then $frac1pi^m+fraca_m-1pi^m-1+cdots+a_0=0$, and multiplying by $pi^m$, we have $pi mid 1$ in $A_pp$, which is a contradiction. Hence $(pi^n)=A_ppcap (pi^n)B_pp$. Note that we used that if $B$ is integral over $A$, then $B_pp$ is integral over $A_pp$. See pages 220-221 of Pete L. Clark's notes.



          I'm not too sure personally which rings are the ones with this property, but hopefully this is helpful.






          share|cite|improve this answer




















            Your Answer





            StackExchange.ifUsing("editor", function ()
            return StackExchange.using("mathjaxEditing", function ()
            StackExchange.MarkdownEditor.creationCallbacks.add(function (editor, postfix)
            StackExchange.mathjaxEditing.prepareWmdForMathJax(editor, postfix, [["$", "$"], ["\\(","\\)"]]);
            );
            );
            , "mathjax-editing");

            StackExchange.ready(function()
            var channelOptions =
            tags: "".split(" "),
            id: "69"
            ;
            initTagRenderer("".split(" "), "".split(" "), channelOptions);

            StackExchange.using("externalEditor", function()
            // Have to fire editor after snippets, if snippets enabled
            if (StackExchange.settings.snippets.snippetsEnabled)
            StackExchange.using("snippets", function()
            createEditor();
            );

            else
            createEditor();

            );

            function createEditor()
            StackExchange.prepareEditor(
            heartbeatType: 'answer',
            convertImagesToLinks: true,
            noModals: true,
            showLowRepImageUploadWarning: true,
            reputationToPostImages: 10,
            bindNavPrevention: true,
            postfix: "",
            imageUploader:
            brandingHtml: "Powered by u003ca class="icon-imgur-white" href="https://imgur.com/"u003eu003c/au003e",
            contentPolicyHtml: "User contributions licensed under u003ca href="https://creativecommons.org/licenses/by-sa/3.0/"u003ecc by-sa 3.0 with attribution requiredu003c/au003e u003ca href="https://stackoverflow.com/legal/content-policy"u003e(content policy)u003c/au003e",
            allowUrls: true
            ,
            noCode: true, onDemand: true,
            discardSelector: ".discard-answer"
            ,immediatelyShowMarkdownHelp:true
            );



            );













             

            draft saved


            draft discarded


















            StackExchange.ready(
            function ()
            StackExchange.openid.initPostLogin('.new-post-login', 'https%3a%2f%2fmath.stackexchange.com%2fquestions%2f2999021%2fcutting-down-an-ideal%23new-answer', 'question_page');

            );

            Post as a guest















            Required, but never shown

























            2 Answers
            2






            active

            oldest

            votes








            2 Answers
            2






            active

            oldest

            votes









            active

            oldest

            votes






            active

            oldest

            votes








            up vote
            4
            down vote



            accepted










            $$A=Bbb ZsubsetBbb R=B;,;;I:=3Bbb Zimplies IB=IBbb R=Bbb R;,;;textso...$$






            share|cite|improve this answer




















            • Is there a general rule in what rings this statement holds? I know it works in Dedekind-Domains, and if $A = K[X]$ and $B=L[X]$ for two fields $K subset L$ it also works.
              – user9620780
              2 days ago















            up vote
            4
            down vote



            accepted










            $$A=Bbb ZsubsetBbb R=B;,;;I:=3Bbb Zimplies IB=IBbb R=Bbb R;,;;textso...$$






            share|cite|improve this answer




















            • Is there a general rule in what rings this statement holds? I know it works in Dedekind-Domains, and if $A = K[X]$ and $B=L[X]$ for two fields $K subset L$ it also works.
              – user9620780
              2 days ago













            up vote
            4
            down vote



            accepted







            up vote
            4
            down vote



            accepted






            $$A=Bbb ZsubsetBbb R=B;,;;I:=3Bbb Zimplies IB=IBbb R=Bbb R;,;;textso...$$






            share|cite|improve this answer












            $$A=Bbb ZsubsetBbb R=B;,;;I:=3Bbb Zimplies IB=IBbb R=Bbb R;,;;textso...$$







            share|cite|improve this answer












            share|cite|improve this answer



            share|cite|improve this answer










            answered 2 days ago









            DonAntonio

            175k1491224




            175k1491224











            • Is there a general rule in what rings this statement holds? I know it works in Dedekind-Domains, and if $A = K[X]$ and $B=L[X]$ for two fields $K subset L$ it also works.
              – user9620780
              2 days ago

















            • Is there a general rule in what rings this statement holds? I know it works in Dedekind-Domains, and if $A = K[X]$ and $B=L[X]$ for two fields $K subset L$ it also works.
              – user9620780
              2 days ago
















            Is there a general rule in what rings this statement holds? I know it works in Dedekind-Domains, and if $A = K[X]$ and $B=L[X]$ for two fields $K subset L$ it also works.
            – user9620780
            2 days ago





            Is there a general rule in what rings this statement holds? I know it works in Dedekind-Domains, and if $A = K[X]$ and $B=L[X]$ for two fields $K subset L$ it also works.
            – user9620780
            2 days ago











            up vote
            2
            down vote













            DonAntonio gave a good counterexample, I just thought I'd try to make the problem of which rings have this property a bit easier by reducing it to a local question.



            The inclusion map $Ito Acap IB$ is surjective if and only if for all primes $newcommandppmathfrakpppinnewcommandSpecoperatornameSpecSpec A$ the localized map $I_ppto (Acap IB)_pp=A_ppcap (IB)_pp=A_ppcap I_pp B_pp$ is surjective. The first equality is proved here, and the second is true, since if $I$ is an ideal in a commutative ring, $A$, $S$ is a multiplicative set, and $M$ is an $A$ module, it's fairly clear that $S^-1IM subseteq (S^-1I)(S^-1M)$, and conversely if $sum_i fraca_is_ifracm_it_i in (S^-1I)(S^-1M)$, multiplying by $prod_i s_it_i$ gives an element of $IM$, so the original element of $(S^-1I)(S^-1M)$ must have been an element of $S^-1IM$.



            Hence to show that $I=Acap IB$ for all ideals $I$ in $A$ it is sufficient to prove the property locally. I.e., that for all points $pp$ of $Spec A$ and ideals $I_ppsubset A_pp$ we have $I_pp=A_ppcap I_pp B_pp$.



            In fact it suffices to just prove this for the maximal ideals. See page 7 here or any standard text on commutative algebra.



            It's not too hard to show that this property holds for the maximal ideals in a Dedekind domain, (with $B$ integral over $A$ ofc) since $A_pp$ is a DVR (when $pp$ is maximal). Thus we have that the ideals of $A_pp$ are $(pi^n)$ for $pi$ a uniformizer for $A_pp$, and if $pi^n-1in(pi^n)B_pp$, we would have $pi^n-1=pi^n b$ for some $bin B_pp$, or $1=pi b$, but $frac1pi$ cannot possibly be integral over $A_pp$, since then $frac1pi^m+fraca_m-1pi^m-1+cdots+a_0=0$, and multiplying by $pi^m$, we have $pi mid 1$ in $A_pp$, which is a contradiction. Hence $(pi^n)=A_ppcap (pi^n)B_pp$. Note that we used that if $B$ is integral over $A$, then $B_pp$ is integral over $A_pp$. See pages 220-221 of Pete L. Clark's notes.



            I'm not too sure personally which rings are the ones with this property, but hopefully this is helpful.






            share|cite|improve this answer
























              up vote
              2
              down vote













              DonAntonio gave a good counterexample, I just thought I'd try to make the problem of which rings have this property a bit easier by reducing it to a local question.



              The inclusion map $Ito Acap IB$ is surjective if and only if for all primes $newcommandppmathfrakpppinnewcommandSpecoperatornameSpecSpec A$ the localized map $I_ppto (Acap IB)_pp=A_ppcap (IB)_pp=A_ppcap I_pp B_pp$ is surjective. The first equality is proved here, and the second is true, since if $I$ is an ideal in a commutative ring, $A$, $S$ is a multiplicative set, and $M$ is an $A$ module, it's fairly clear that $S^-1IM subseteq (S^-1I)(S^-1M)$, and conversely if $sum_i fraca_is_ifracm_it_i in (S^-1I)(S^-1M)$, multiplying by $prod_i s_it_i$ gives an element of $IM$, so the original element of $(S^-1I)(S^-1M)$ must have been an element of $S^-1IM$.



              Hence to show that $I=Acap IB$ for all ideals $I$ in $A$ it is sufficient to prove the property locally. I.e., that for all points $pp$ of $Spec A$ and ideals $I_ppsubset A_pp$ we have $I_pp=A_ppcap I_pp B_pp$.



              In fact it suffices to just prove this for the maximal ideals. See page 7 here or any standard text on commutative algebra.



              It's not too hard to show that this property holds for the maximal ideals in a Dedekind domain, (with $B$ integral over $A$ ofc) since $A_pp$ is a DVR (when $pp$ is maximal). Thus we have that the ideals of $A_pp$ are $(pi^n)$ for $pi$ a uniformizer for $A_pp$, and if $pi^n-1in(pi^n)B_pp$, we would have $pi^n-1=pi^n b$ for some $bin B_pp$, or $1=pi b$, but $frac1pi$ cannot possibly be integral over $A_pp$, since then $frac1pi^m+fraca_m-1pi^m-1+cdots+a_0=0$, and multiplying by $pi^m$, we have $pi mid 1$ in $A_pp$, which is a contradiction. Hence $(pi^n)=A_ppcap (pi^n)B_pp$. Note that we used that if $B$ is integral over $A$, then $B_pp$ is integral over $A_pp$. See pages 220-221 of Pete L. Clark's notes.



              I'm not too sure personally which rings are the ones with this property, but hopefully this is helpful.






              share|cite|improve this answer






















                up vote
                2
                down vote










                up vote
                2
                down vote









                DonAntonio gave a good counterexample, I just thought I'd try to make the problem of which rings have this property a bit easier by reducing it to a local question.



                The inclusion map $Ito Acap IB$ is surjective if and only if for all primes $newcommandppmathfrakpppinnewcommandSpecoperatornameSpecSpec A$ the localized map $I_ppto (Acap IB)_pp=A_ppcap (IB)_pp=A_ppcap I_pp B_pp$ is surjective. The first equality is proved here, and the second is true, since if $I$ is an ideal in a commutative ring, $A$, $S$ is a multiplicative set, and $M$ is an $A$ module, it's fairly clear that $S^-1IM subseteq (S^-1I)(S^-1M)$, and conversely if $sum_i fraca_is_ifracm_it_i in (S^-1I)(S^-1M)$, multiplying by $prod_i s_it_i$ gives an element of $IM$, so the original element of $(S^-1I)(S^-1M)$ must have been an element of $S^-1IM$.



                Hence to show that $I=Acap IB$ for all ideals $I$ in $A$ it is sufficient to prove the property locally. I.e., that for all points $pp$ of $Spec A$ and ideals $I_ppsubset A_pp$ we have $I_pp=A_ppcap I_pp B_pp$.



                In fact it suffices to just prove this for the maximal ideals. See page 7 here or any standard text on commutative algebra.



                It's not too hard to show that this property holds for the maximal ideals in a Dedekind domain, (with $B$ integral over $A$ ofc) since $A_pp$ is a DVR (when $pp$ is maximal). Thus we have that the ideals of $A_pp$ are $(pi^n)$ for $pi$ a uniformizer for $A_pp$, and if $pi^n-1in(pi^n)B_pp$, we would have $pi^n-1=pi^n b$ for some $bin B_pp$, or $1=pi b$, but $frac1pi$ cannot possibly be integral over $A_pp$, since then $frac1pi^m+fraca_m-1pi^m-1+cdots+a_0=0$, and multiplying by $pi^m$, we have $pi mid 1$ in $A_pp$, which is a contradiction. Hence $(pi^n)=A_ppcap (pi^n)B_pp$. Note that we used that if $B$ is integral over $A$, then $B_pp$ is integral over $A_pp$. See pages 220-221 of Pete L. Clark's notes.



                I'm not too sure personally which rings are the ones with this property, but hopefully this is helpful.






                share|cite|improve this answer












                DonAntonio gave a good counterexample, I just thought I'd try to make the problem of which rings have this property a bit easier by reducing it to a local question.



                The inclusion map $Ito Acap IB$ is surjective if and only if for all primes $newcommandppmathfrakpppinnewcommandSpecoperatornameSpecSpec A$ the localized map $I_ppto (Acap IB)_pp=A_ppcap (IB)_pp=A_ppcap I_pp B_pp$ is surjective. The first equality is proved here, and the second is true, since if $I$ is an ideal in a commutative ring, $A$, $S$ is a multiplicative set, and $M$ is an $A$ module, it's fairly clear that $S^-1IM subseteq (S^-1I)(S^-1M)$, and conversely if $sum_i fraca_is_ifracm_it_i in (S^-1I)(S^-1M)$, multiplying by $prod_i s_it_i$ gives an element of $IM$, so the original element of $(S^-1I)(S^-1M)$ must have been an element of $S^-1IM$.



                Hence to show that $I=Acap IB$ for all ideals $I$ in $A$ it is sufficient to prove the property locally. I.e., that for all points $pp$ of $Spec A$ and ideals $I_ppsubset A_pp$ we have $I_pp=A_ppcap I_pp B_pp$.



                In fact it suffices to just prove this for the maximal ideals. See page 7 here or any standard text on commutative algebra.



                It's not too hard to show that this property holds for the maximal ideals in a Dedekind domain, (with $B$ integral over $A$ ofc) since $A_pp$ is a DVR (when $pp$ is maximal). Thus we have that the ideals of $A_pp$ are $(pi^n)$ for $pi$ a uniformizer for $A_pp$, and if $pi^n-1in(pi^n)B_pp$, we would have $pi^n-1=pi^n b$ for some $bin B_pp$, or $1=pi b$, but $frac1pi$ cannot possibly be integral over $A_pp$, since then $frac1pi^m+fraca_m-1pi^m-1+cdots+a_0=0$, and multiplying by $pi^m$, we have $pi mid 1$ in $A_pp$, which is a contradiction. Hence $(pi^n)=A_ppcap (pi^n)B_pp$. Note that we used that if $B$ is integral over $A$, then $B_pp$ is integral over $A_pp$. See pages 220-221 of Pete L. Clark's notes.



                I'm not too sure personally which rings are the ones with this property, but hopefully this is helpful.







                share|cite|improve this answer












                share|cite|improve this answer



                share|cite|improve this answer










                answered yesterday









                jgon

                9,50911538




                9,50911538



























                     

                    draft saved


                    draft discarded















































                     


                    draft saved


                    draft discarded














                    StackExchange.ready(
                    function ()
                    StackExchange.openid.initPostLogin('.new-post-login', 'https%3a%2f%2fmath.stackexchange.com%2fquestions%2f2999021%2fcutting-down-an-ideal%23new-answer', 'question_page');

                    );

                    Post as a guest















                    Required, but never shown





















































                    Required, but never shown














                    Required, but never shown












                    Required, but never shown







                    Required, but never shown

































                    Required, but never shown














                    Required, but never shown












                    Required, but never shown







                    Required, but never shown






                    Popular posts from this blog

                    How to check contact read email or not when send email to Individual?

                    Bahrain

                    Postfix configuration issue with fips on centos 7; mailgun relay